Class #4

Ace your homework & exams now with Quizwiz!

savings amount

the expressed dollar value that a person is saving

Pauline, age 63, is an employee of C Corp who earns $140,000 per year. She has already paid off her mortgage. She currently saves 10% of her salary in her 401(k) as well as $5,000 per year into a Traditional IRA. Based on the information above, what do you expect Pauline's wage replacement ratio (WRR) to be at retirement?

$110,290 ÷ $140,000 = 78.78% WRR (includes 7.65% payroll taxes)

Theresa, a self-employed dentist, currently earns $100,000 per year. Theresa has always been a self-proclaimed saver, and she saves 25 percent per year of her Schedule C net income. Assume Theresa pays $13,000 in Social Security taxes. Theresa plans to pay off her home mortgage at retirement and live debt-free. She currently spends $25,000 per year on her mortgage. Based on the information above, what do you expect Theresa's wage replacement ratio (WRR) to be at retirement?

$37,000 ÷ $100,000 = 37.00% WRR

Assume Anna and Bart have the same amount of income and the same expenses and both are about to retire. They both want to determine their appropriate WRR. Anna's house and car will be paid off before retirement, while Bart will continue to have to make mortgage payments and car loan payments in retirement. They both have other expenses that will decrease by the same amount. Below is the bottom-up approach to calculating each WRR.

Bottom up Approach Example

80% of men past the age of 65 work. True or False?

False

Sheryl makes $100,000 per year and is about to retire. She has the following expenses that will be eliminated when she retires: House payment (annual) - $30,000 (the mortgage will be paid off when she retires) Savings into her 401(k) plan - $24,000 Commuting costs to work - $5,000 (includes tolls, gas parking, etc.) Work clothes per year - $2,000 Lunches & coffee at work - $2,000 FICA payments - $7,650 The total of these expenses that will be eliminated equals $70,650. Assume that she has other expenses that might increase, such as travel, which increase her expenses from $29,350 to $40,000.

If that were the case, Sheryl's wage replacement ratio needed equals 40% ($40,000 divided by $100,000), which is dramatically different than the commonly cited 70% to 80% WRR.

Work period that remains at a given point in time before retirement. The number of years a client has to save for retirement. The average retirement age is between 62 and 64.

Remaining Work Life Expectancy (RWLE)

Time period beginning at retirement and ending at death. Time period planned for by the financial planner and the retiree during the work life.

Retirement Life Expectancy (RLE)

Samuel, aged 47, earns $400,000 annually from Atlas, which sponsors a SIMPLE, and the makes the required 2% non-elective contribution to all eligible employees. What is the maximum total contribution to Samuel's account in 2021, including both employee and employer contributions? A) $16,500 B) $19,300 C) $19,500 D) $26,000

The correct answer is (B). Samuel can defer up to $13,500 for 2021. The non-elective contribution equals 2 percent of his salary limited to the 2021 limit of $290,000, which is $5,800. Thus, the total equals $19,200.

Marleen, who is 37 years old, is an employee of Zcrypt, Inc. (Zcrypt). Zcrypt sponsors a SEP IRA and would like to contribute the maximum amount to Marleen's account for the plan year. If Marleen earns $32,000 per year, what is the maximum contribution Zcrypt can make on her behalf to the SEP IRA? A) $8,000 B) $19,000 C) $32,000 D) $57,000

The correct answer is (A). Contributions to a SEP IRA are limited to the lesser of 25 percent of the employee's compensation or $61,000 (2022). In this case, Marleen's compensation is $32,000, so the contribution on her behalf would be limited to 25 percent of $32,000, or $8,000.

All of the following plans are permitted to offer a Roth account as part of the plan EXCEPT: A) A private 457(b) plan B) A governmental 457(b) plan C) A 403(b) plan D) A 401(k) plan

The correct answer is (A). Private 457(b) plans cannot offer a Roth account. The other plans are permitted to do so.

Dan owns and operates Schoepf's Sales Solutions (3S), a sole proprietorship. 3S sponsors a profit-sharing plan. During the year, Dan had net income of $250,000 and half his self-employment taxes were $11,000. Assuming Dan is over the age of 50, what is the maximum amount that Dan can contribute to the profit-sharing plan on his behalf this year? A) $47,800 B) $54,300 C) $61,000 D) $67,500

The correct answer is (A). $250,000 Net Income ($11,000) Less ½ SE Tax ____________________________$239,000 Net SE Income × 0.25 ÷ 1.25 $47,800 The fact that Dan is over the age of 50 is not relevant. Only employees contribute to CODAs. Catch-up contributions are not made by employers.

A contributory plan is one in which I. employees are permitted to make contributions to the plan .II. employers are required to make contributions to the plan. A) I only B) II only C) Both I and II D) Neither I nor II

The correct answer is (A). A contributory plan is a plan that allows the plan participant (the employee) to contribute to the plan. Employers are not required to contribute to contributory plans.

All of the following plans require mandatory funding EXCEPT A) a profit-sharing plan. B) a defined-benefit pension plan. C) a cash-balance pension plan. D) a money-purchase pension plan (integrated).

The correct answer is (A). A profit-sharing plan does not require mandatory funding. All of the other listed plans do.

Seth operates a small business with his wife, Morgan. They have no other employees. They have $340,000 of assets in the plan. Which of the following forms must they file? A) Form 5500 EZ B) Form 5500 SF C) Form 5500 D) They do not have to file form 5500.

The correct answer is (A). Because the plan only covers the owner and spouse, they can file form 5500 EZ.

Robin works for a small publishing company. The company built a 2,500-square-foot gym on its premises. The cost of the facility was $80,000. The cost of a comparable membership to a local gym is $60/month. Robin, her husband, and her two children all use the gym regularly. How much, if anything, must Robin include in her yearly gross income related to this fringe benefit? A) $0 B) $720 C) $2,160 D) $2,880

The correct answer is (A). Robin does not have to include any income for the use of the athletic facilities; the exclusion for payment of health club facility dues is only provided when the facilities are on the employer's business premises and are solely for the use of the employees and their family members. Thus, this use qualifies. TCJA 2017 did not change this provision.

Which of the following are correct regarding 403(b) plans? I. Employee elective deferrals may be contributed to a Roth account within a 403(b) plan.II. 403(b) plans must comply with the ADP and ACP tests.III. An employee of the New Orleans Museum of Art (a not-for-profit) could defer more than $27,500 into the Museum's 403(b) plan. A) I only B) I and II C) I and III D) II and III

The correct answer is (A). Statement II is incorrect as 403(b) plans do not have to comply with the ADP test.Statement III is not correct as the only way to exceed $26,000 (2021) would be to make use of the additional catch-up contribution, which would not be available to an employee of a museum — only to employees of health care, education, or religious organizations.

In January, Don, who is 75 years old, confirmed in an email with his financial advisor that he wanted to take a distribution of $50,000 from his IRA and roll it over into a new IRA. His financial advisor inadvertently moved the funds into a taxable account. The advisor discovered this mistake at the end of the year and corrected it. What will be the result of these transactions? A) The distribution will be considered taxable income unless Don receives a waiver of the 60-day rollover requirement from the IRS or uses the self-certification process. B) The distribution will be considered taxable income. Don must pay the income tax but may seek relief from the financial advisor. C) Because the advisor corrected his mistake before the end of the calendar year, the distribution will not be considered taxable income. D) Because Don is older than age 72, the distribution will not be considered taxable income.

The correct answer is (A). The IRS regularly grants waivers of the 60-day rollover period in cases of a mistake made by a financial advisor. The taxpayer may also qualify to use the self-certification process (see Rev. Proc. 2016-2017).

Krista has worked for GT for the last 20 years and has been a participant in its defined-benefit plan. In the last 10 years, her salary has changed significantly. Over the last 10 years, her compensation was as follows: Year 1: $190,000 Year 2: $100,000 Year 3: $120,000 Year 4: $100,000 Year 5: $240,000 Year 6: $200,000 Year 7: $160,000 Year 8: $180,000 Year 9: $150,000 Year 10: $110,000 In 2022, what is the most that she could receive as a pension payment? A) $200,000 B) $245,000 C) $246,667 D) $280,000

The correct answer is (A). The maximum distribution from a defined-benefit plan in 2022 is the lesser of $245,000 or the average of the 3 highest consecutive years of compensation. The average of Krista's 3 highest consecutive years of compensation, $240,000, $200,000, and $160,000, equals $200,000.

Which of the following correctly describes the shared-payment approach used in some qualified domestic relations orders? A) It is used to split pension payments when plan participants are already receiving payments. B) It is typically used to assign plan benefits before annuity election. C) It is used to share the taxation burden of qualified plan distributions between the participant and a third party. D) Under this approach, it is not required that the participant and third party both receive a payment in any given year.

The correct answer is (A). The shared-payment approach splits payments between a participant and third party when distributions are already being made from a pension annuity. Answer (B) describes the separate interest method. Answer (C) and (D) are incorrect statements.

Janet works for IBM, a publicly traded company that sponsors a stock bonus plan. All of the following statements regarding the plan are correct EXCEPT: A) If Janet has less than 3 years of service, she is permitted to diversify one-half of company match contributions that consist of IBM stock. B) Janet is permitted to vote the shares of IBM within her account. C) Upon termination, Janet must be given the right to receive IBM stock held in the plan as part of a distribution. D) If the distribution of IBM stock is made to Janet in installments over a 2-year period, the fair market value of all employer securities distributed in the installment distribution will be taxable as ordinary income.

The correct answer is (A).After 3 years of service, employer contributions of company stock can be fully diversified. Before 3 years, employees do not have the right to diversify company stock in a stock bonus plan of a publicly traded company. The other statements are correct.

ABC, Inc., sponsors a defined-contribution plan. Seth, age 39, has compensation of $150,000 for the year. ABC has made a $20,000 profit-sharing plan contribution for Seth, and $5,000 of plan forfeitures were allocated to Seth's profit-sharing plan during the year. How much can Seth defer into his CODA plan (401(k) plan) for 2022? A) $20,500 B) $25,000 C) $31,000 D) $36,000

The correct answer is (A).The annual-additions limit permits $61,000 (2022) of contributions to be made on Seth's behalf to qualified plans for the year. The employee elective deferral limit is $20,500 for 2022 and is included in the $61,000 calculation. In this case, Seth received a total contribution of $25,000 on his behalf into the plan, so his deferral limit is not limited because of the other calculations and Seth can defer the maximum, or $20,500, to the plan for 2022.

The beneficiary(ies) of a key-person life insurance policy generally is (are) I. the employerII. the key person's spouse or dependents A) I only B) II only C) Both I and II D) Neither I nor II

The correct answer is (A).The beneficiary is usually the company because the company has suffered the loss of the key person. The key person's family would need to purchase a separate life insurance policy.

Generally, distributions from a retirement plan are subject to income tax as ordinary income. All of the following tax treatments are exceptions to ordinary income on a distribution from a qualified plan EXCEPT: A) Pre-1974 capital-gain treatment B) Nonrecognition of gain treatment C) 10-year forward averaging D) Net unrealized appreciation

The correct answer is (B). Nonrecognition of gain treatment is the deferral of capital gain (not ordinary income) that has nothing to do with a distribution from a qualified plan.

Philip has a vested account balance in his employer-sponsored qualified money-purchase pension plan of $60,000. He has 2 years of service with his employer, and the plan follows the least generous graduated vesting schedule permitted under the Pension Protection Act of 2006 (PPA 2006). Philip has an outstanding loan balance of $15,000 on a $20,000 loan taken within the prior 12 months. What is the maximum loan Philip could take from this qualified plan, assuming the plan permits loans? A) $15,000 B) $30,000 C) $35,000 D) $50,000

The correct answer is (A).The maximum loan an individual can take is the lesser of $50,000 or 50 percent of his or her vested account balance. In this case, Philip has a vested account balance of $60,000, so the maximum loan would be 50 percent of $60,000, or $30,000. However, since Philip had an outstanding loan balance of $15,000 within 12 months, the maximum loan available must be reduced by $15,000. In this case, the maximum loan Philip could take from the qualified plan is $15,000 (or $30,000 − $15,000).

Acme, Inc., has 200 total employees, 150 of which are non-excludable employees. Ten employees are highly compensated. Seven of the 10 highly compensated and 100 of the 140 non-highly compensated employees are covered under Acme's qualified plan. The average accrued benefits for the highly compensated employees is 3 percent, and the average accrued benefit for the non-highly compensated is 1.5 percent. Which of the following statements is true regarding coverage? I. The plan passes the ratio percentage test.II. The plan passes the average benefits test. A) I only B) II only C) Both I and II D) Neither I nor II

The correct answer is (A).The ratio percentage test compares the percent of non-highly compensated to the percent of highly compensated covered employees. The ratio must be greater than or equal to 70 percent for the plan to pass the ratio percentage test. The calculation for Acme's qualified plan is as follows: NHC = 100/140 = 71.43% HC = 7/10 = 70% 71.43%/70% = 102% Acme's plan passes the ratio percentage test requirement of 70 percent.The average benefits test requires the average benefit of the non-highly compensated employees to be at least 70 percent of the average benefit of the highly compensated. Acme's plan does not satisfy the average benefits test, because the average benefit of the non-highly compensated compared to the average benefit of the highly compensated is less than 70 percent (that is, 1.5% ÷ 3% = 50%) (Fail).

Crystal, a waitress at Hot Dog Heaven, works from 7 a.m. to 4 p.m. five days a week. Each workday she is furnished, without charge, two meals, valued at $10 each. The manager of Hot Dog Heaven encourages her to eat breakfast in the employee breakroom each day before 7 a.m. but does not expressly require her to do so. The manager does, however, require her to eat her lunch in the employee breakroom. How much of the value of her meals does she have to include in her gross income per day? A) $0 B) $10 C) $15 D) $20

The correct answer is (A).These meals are provided for the convenience of the employer. The changes under TCJA 2017 should not impact this scenario as this likely falls into the category of de minimis.

IRC §132(e) defines a fringe benefit as any property or service provided to an employee by an employer that is so small in value that it makes accounting for it unreasonable or administratively impracticable when taking into account the frequency with which similar fringe benefits are provided by the employer to employees. What type of fringe benefit is this? A) De minimis B) Working-condition fringe C) Employee benefit D) No-additional-cost fringe

The correct answer is (A).This is the definition of a de minimis fringe benefit.

Jordan owns and operates Jordan's Exotic Journeys (JEJ), a sole proprietorship. JEJ sponsors a profit-sharing plan. During the year, Jordan had net income of $150,000 and half of her self-employment taxes were $10,000. She has decided to make a 15 percent contribution for her employees for the year. Assuming Jordan is over the age of 50, what amount will she contribute for herself to the plan this year? A) $16,957 B) $18,261 C) $28,000 D) $56,000

The correct answer is (B). $150,000 Net Income ($10,000) Less ½ SE Tax $140,000 Net SE Income × 0.130435 0.15 ÷ 1.15 $18,261

Which of the following vesting schedules may a top-heavy qualified cash-balance plan use? A) 2-to-6 year, graduated B) 3 year, cliff C) 3-to-7 year, graduated D) 5 year, cliff

The correct answer is (B). As a result of the Pension Protection Act of 2006, cash-balance plans must vest at least as fast as a 3-year cliff vesting schedule. They cannot use the 2-to-6 year graduated schedule.

Blake was awarded 1,000 shares of restricted stock of Acme Corporation at a time when the stock price was $7. Assume Blake properly makes an 83(b) election at the date of the award. The stock vests 3 years later at a price of $19 and Blake sells it then. What are Blake's tax consequences in the year he sells the stock? A) Blake has W-2 income of $19,000. B) Blake has a long-term capital gain of $12,000. C) Blake has W-2 income of $12,000. D) Blake has a $19,000 long-term capital gain.

The correct answer is (B). At the time Blake makes the 83(b) election, the value of the stock at that date will be included in his taxable income. Thus, Blake will have W-2 income of $7,000 ($7 × $1,000) in the year the stock was awarded. Any gain beyond that will be a capital gain. Thus, the $12,000 increase will be capital gain.

What is the earliest age when a participant can make an IRA catch-up contribution? A) 45 B) 50 C) 60 D) 65

The correct answer is (B). Catch-up contributions are permitted for those who have attained the age of 50.

XYZ covered the following employees under its qualified plan: I. Rob, a 4 percent owner and employee with compensation of $32,000 II. Robin, Rob's cousin, who is a commissioned salesperson with compensation of $160,000 last year (the highest paid employee) and who owns 2 percent of the company stock III. Randy, the chief operating officer, who had compensation of $155,000 last year but was not in the top 20 percent of paid employees Assuming the company made the 20 percent election when determining who is highly compensated, which of the following statements is correct about this company in 2022? A) Rob and Robin are both key employees. B) Robin is a key employee, but Rob is not. C) Randy is a key employee but not a highly compensated employee. D) None of these people are highly compensated employees or key employees.

The correct answer is (B). For 2022, a key employee is an employee who at any time during the plan year or prior year met one of the following definitions: A greater-than-5-percent owner A greater-than-1-percent owner with compensation that is greater than $150,000 (not indexed) An officer with compensation in excess of $200,000 For 2022, highly compensated employees are employees who are one of the following: A more-than-5-percent owner at any time during the plan year or preceding plan year An employee with compensation in excess of $135,000 for the prior plan year and who, if elected, is in the top 20 percent of paid employees ranked as to compensation Rob is not a 5 percent owner. His compensation is too low for him to be highly compensated. Robin is not a 5 percent owner. Attribution does not include cousins. She meets the definition of highly compensated because her income is over $135,000 and she is in the top 20 percent of paid employees. She is also a key employee as she is a greater-than-1-percent owner and has compensation in excess of $150,000. Randy is an officer but not an owner. However, he does not meet the definition of highly compensated because he is not in the top 20 percent of paid employees. He is not a key employee because his compensation is less than the annual limit.

SEPs and SIMPLEs are tax-advantaged retirement plans that are less complex than qualified plans and easier to implement. Which of the following statements is (are) true regarding SEPs and SIMPLEs? I. For an employee with a salary of $30,000, more money can be contributed (from both employee and employer) to a SEP than a SIMPLE. II. The vesting and distribution rules for both plans are almost identical. A) I only B) II only C) Both I and II D) Neither I nor II

The correct answer is (B). Statement I is not correct as the deferral into a SIMPLE is greater than 25 percent of the $30,000 salary, without regard to a catch-up and/or a match by the employer, while the deferral to the SEP would be 25 percent of the employee's salary. Statement II is correct. SIMPLEs do have the extra penalty for 2 years, but other than that they are the same.

Tidewater Company has 1,000 eligible employees and sponsors a defined-benefit pension plan. The company is unsure if it is meeting all of its testing requirements. What is the minimum number of employees that must be covered by Tidewater's defined-benefit pension plan for the plan to conform with ERISA? A) 40 B) 50 C) 400 D) 500

The correct answer is (B). The 50/40 rule requires that defined-benefit plans cover the lesser of 50 employees or 40 percent of all eligible employees. In this example, 50 employees is less than 40 percent of 1,000, or 400 employees.

Thomas, who is 49 years old, received a distribution from his Roth account of his employer's 401(k) plan in the amount of $100,000 on August 11. He has been a participant in the plan for 10 years. His adjusted basis in the plan was $600,000, and the fair market value of the account as of August 11 was $1 million. The purpose of the distribution was to buy a Porsche for himself for his birthday. What is the taxable amount of the distribution and any applicable penalty? A) $0 taxable, $0 penalty because it is a qualified distribution B) $40,000 taxable, $4,000 tax penalty C) $40,000 taxable, $0 tax penalty D) $100,000 taxable, $10,000 tax penalty

The correct answer is (B). The distribution is not a qualified distribution because he is not aged 59½, disabled, or dead. Thus, the pro rata portion of the distribution that relates to the earnings is taxable─40 percent in this case. The penalty is 10 percent of $40,000, or $4,000.

Kay turned 72 on March 17 of Year 2. Her profit-sharing account balance was $500,000 at the end of Year 1 and $550,000 at the end of Year 2. Her beneficiary is her granddaughter, Jordan, who turned 12 years old on July 23 of Year 2. Assume that the joint life expectancy factor for a 71-year-old and a 12-year-old is 71 and the joint life expectancy for a 72-year-old and a 13-year-old is 70. Also, assume that the life expectancy factor based on the uniform lifetime table for someone who is 70, 71, and 72, is 27.4, 26.5, and 25.6, respectively. What is Kay's minimum distribution for Year 2? A) $18,248 B) $19,532 C) $20,073 D) $20,755

The correct answer is (B). Year 2's required minimum distribution for Kay is $19,532 (or $500,000 divided by 25.6) because she is 72 years old as of December 31 of Year 2. Her beneficiary's age is not relevant.

Monica, aged 40, receives $150,000 of group term life insurance from her employer as an employee benefit. The employer pays the premium equal to $300 for Monica. As a result, what will be Monica's taxable income? Assume the cost of protection per $1,000 of protection for one month is $0.10 for employees ages 40 through 44. A) $0 (a non-taxable benefit) B) $120 (40% of the premium) C) $200 (2/3 of the premium) D) $300 (the full premium)

The correct answer is (B).$0.10 × 12 months = $1.20 per thousand × $100,000, (the excess over $50,000 group term) = $120.00.

A cross-purchase buy-sell agreement is most appropriate for which of the following organizations? A) A sole proprietorship B) A partnership with 3 partners C) A partnership with 18 partners D) A large C-corporation

The correct answer is (B).A cross-purchase buy-sell agreement is most appropriate for a small partnership. For larger organizations, the number of individual life insurance policies required to create a cross-purchase buy-sell agreement makes them too complex and cost-prohibitive. In those cases, an entity-purchase buy-sell agreement would be more appropriate.

All of the following are correct in regard to qualified plan terminations EXCEPT: A) A 20 percent or 50 percent excise tax applies on assets a plan sponsor recoups from a terminated qualified plan. B) Any outstanding liabilities or obligations of the plan are waived upon plan termination. C) A plan sponsor is generally not allowed to withdraw assets from a terminating plan. D) It is simpler for a plan sponsor to terminate a defined-contribution plan than a defined-benefit plan.

The correct answer is (B).Any outstanding liabilities or obligations must first be paid off by plan assets before a plan sponsor can recoup those assets. All of the other statements are correct.

Employee Age Covered Comp. Employee Deferral Years in Plan Bob 57 $160,000 10% 15 Lucy 23 $100,000 Not yet participating N/A Georgina 37 $80,000 20% 15 David 31 $76,000 4% 2 Bob has employed his son, David, and his new wife, Lucy. Bob feels the newlyweds are spendthrifts, and he wants the 401(k) plan to encourage them to save money and stick with their employment. He asks for your help in maximizing plan contributions, even if that entails making alterations in the plan. Which of the following would NOT be one of your recommendations? A) To maximize their deferrals, Bob and David should contribute more. B) Lucy should enter the plan and contribute 50% of her salary. C) Bob could add a profit-sharing contribution as a means of increasing contributions to the plan. D) In order to fully vest in the plan, David should be encouraged to stay employed with the Bagelry.

The correct answer is (B).Lucy should enter the plan, but she would not be able to contribute 50 percent because the limit for a 401(k) contributions is significantly lower than $50,000. David and Bob should both increase their contributions - Bob because he is below the maximum and because of the $6,500 catch-up contribution allowed for his age. The graduated vesting schedule means David is not 100 percent vested and should be encouraged to stay with the shop. The family is contributing far less than the annual limit for profit sharing plans, so Bob can further increase the contribution for all in the family by adding a profit-sharing contribution to the plan.

Profit-sharing and stock-bonus plans have similarities and differences. Which of the following statements regarding these plans are correct? I. Both plans allow participants to vote their shares that are held by the plan.II. Stock-bonus and profit-sharing plans can make lump-sum distributions of employer securities at the termination of an employee. A) I only B) II only C) Both I and II D) Neither I nor II

The correct answer is (B).Only participants in a stock bonus plan can vote their shares. Both plans can distribute employer stock.

Which of the following plans requires an actuary on an ongoing basis? A) A tandem plan B) A cash-balance plan C) A 412(e) plan D) A target-benefit plan

The correct answer is (B).The cash-balance plan needs an actuary. The tandem plan consists of a money-purchase pension plan and a profit-sharing plan, so it does not need an actuary. The 412(e) plan is fully funded with insurance and does not need an actuary. The target-benefit plan does not need an actuary after it is set up.

If an employee earning $60,000 had access to multiple plans, which combination would allow for the largest contribution, including both employer and employee contributions? A) A 401(k) plan and a profit-sharing plan B) A governmental 457(b) plan and a 403(b) plan C) A profit-sharing plan and a SEP D) A 401(k) plan and a 403(b) plan

The correct answer is (B).The employee could defer $19,500 to both the 457 plan and the 403(b) plan for a total of $39,000 in 2021. That amount, without considering any matching contributions, is larger than the other combinations. The reason for this is that the "one deferral per year rule" that applies to 401(k) plans and 403(b) plans does not apply to 457 plans.

An employee just received a non-qualified stock option with an exercise price of $50. The fair market value of the stock on the date of the grant is also $50. At the date of the grant, what is the tax treatment of the option? A) The employee must pay ordinary income taxes on the $50. B) The employee will not owe any taxes on the date of the grant. C) The employee must pay capital gains taxes on the $50. D) If the stock price increases to $55 by the end of the year in which the option is granted, the employee must pay capital gains taxes on the $5 gain.

The correct answer is (B).The employee will not have any taxable income at the date of the grant provided the exercise price is equal to the fair market value of the stock.

James retires after 50 years of working for Zacker Industries. His boss gives him a $400 Apple watch as a retirement gift. Is it taxable to James? A) No. Retirement gifts are not taxable if they are under $1,600. B) No. It does not exceed the $400 limit. C) Yes. All gifts from employers are considered compensation. D) Yes. Gifts given without a qualified award plan are taxable.

The correct answer is (B).The value of the gift is relatively small ($400 or smaller) and, thus, not included. The TCJA 2017 requires the gift to be tangible personal property. The Apple watch certainly qualifies as tangible personal property.

All of the following people would be considered a highly compensated employee for 2022 EXCEPT A) Amy, a 2 percent owner whose salary was $165,000 last year B) Red, a 6 percent owner whose salary was $23,500 for the last 5 years C) Reese, an officer who earned $115,000 last year and is the sixth highest paid employee of 96 employees D) Hank, a 0.5 percent owner who earned $136,000 last year and is in the top 20 percent of paid employees

The correct answer is (C). Amy and Hank are highly compensated due to their compensation being greater than $135,000 for 2022. Red is HC because he is a greater-than-5-percent owner. Reese is not highly compensated because she does not have compensation greater than $135,000.

Which of the following is an example of a qualified retirement plan? A) A cash or deferred arrangement (CODA) B) A public 457(b) plan C) An employee stock ownership plan (ESOP) D) An incentive stock option (ISO) plan

The correct answer is (C). An ESOP is the only qualified plan. A CODA is an arrangement that attaches to a profit-sharing plan. A 457 plan and an ISO plan are not qualified plans.

Which of the following are acceptable reasons for an employer to terminate a qualified retirement plan? I. The employer is not profitable and cannot afford to make plan contributions.II. The employer wants to reduce the cost of retirement benefits. As a result, the employer terminates a defined-benefit plan and replaces it with a 401(k) plan. A) I only B) II only C) Both I and II D) Neither I nor II

The correct answer is (C). Both statements are correct.

Which of the following statements regarding required notifications is correct? I. Employers are required to provide a summary plan description, that is, a summary of the details of the qualified retirement plan, to employees, participants, and beneficiaries receiving benefits within 90 days after the person becomes a participant.II. Employers are required to provide the plan participants notices of any plan amendments or changes within 210 days after the end of the plan year in which a change is adopted and applies when there are substantive changes in the plan. A) I only B) II only C) Both I and II D) Neither I nor II

The correct answer is (C). Both statements are correct.

Qualified plans must satisfy many tests to maintain qualified status. Which of the following is correct regarding coverage tests? A) Defined-benefit (DB) plans must pass any two of the four coverage tests. B) Coverage testing never includes leased employees as part of the calculation. C) Stock bonus plans must satisfy one of three coverage tests only. D) Employees who do not meet the eligibility requirements are still included in the determination of at least one of the coverage tests.

The correct answer is (C). Options (A), (B), and (D) are incorrect. Qualified plans must pass one of the three coverage tests, and DB plans must pass one of the three tests and the 50/40 test. The 50/40 test is based on total employees, not necessarily non-highly compensated employees. All coverage tests exclude non-eligible employees from the calculation. Leased employees may have to be included in the coverage calculation if they meet certain requirements.

Assuming the participant has not yet attained age 59½, all of the following distributions from a profit-sharing plan would not be subject to the 10 percent early withdrawal penalty EXCEPT A) a distribution made from an IRA that was established for a 50-year-old former spouse under a qualified domestic relations order (QDRO) in the profit-sharing plan pursuant to a divorce from the participant. B) a distribution from the plan to a participant to pay for medical expenses. C) a distribution to pay for costs of higher education for a participant's 18-year-old daughter. D) a distribution made to the participant after she separated from service at age 57.

The correct answer is (C). Statements (A), (B), and (D) are not subject to the 10 percent early withdrawal penalty. Statement (C) is subject to the 10 percent early withdrawal penalty, but would not be if the distribution was instead made from an IRA.

Jacques, who is 52 years old, works for NYU and is a participant in NYU's tax-sheltered annuity (TSA) program. How much could he defer in the TSA in 2021 if he has a salary of $121,000? A) $13,500 B) $16,000 C) $19,500 D) $26,000

The correct answer is (D). The deferral for a 403(b) plan or TSA is $19,500 for 2021. There is an additional catch-up contribution of $6,500 that is allowed for individuals who have attained age 50.

John is a consultant who issues an invoice on December 18. The company he works for always pays within ten days, including the time it takes for the check to be mailed. If John checks his mailbox on January 1 and the check is there, which of the following tax rules would cause it to be taxable in the prior year? A) Substantial risk of forfeiture B) Tax triads C) Constructive receipt D) Economic benefit doctrine

The correct answer is (C). Substantial risk of forfeiture deals with a risk of a payment not being made and its impact on taxability. Tax triads do not have anything to do with deferred compensation. The economic benefit doctrine provides that an employee will be taxed on funds or property set aside for the employee if the funds or property are unrestricted and non-forfeitable.

Mindy, who is 44 years old, has four IRAs: three are traditional IRAs and one is a Roth IRA. On January 12, she converts $40,000 from one traditional IRA to the Roth IRA. On May 5, she takes a distribution of $20,000 from another traditional IRA and deposits the full $20,000 into her checking account. On May 20, she deposits $20,000 into the third traditional IRA. Which of the following statements is correct? A) Mindy owes tax and a penalty on $40,000. B) Mindy owes tax and a penalty on the $20,000 because it represents her second rollover during the year. C) The $20,000 rollover qualifies for the 60-day rollover exception. D) Mindy owes ordinary income tax on $60,000, but she does not owe a penalty.

The correct answer is (C). The first distribution is a Roth conversion and is subject to income tax, but it is not subject to a penalty. The second distribution qualifies for the 60-day rollover exception, which means it is not subject to income tax or penalty.

Jamal has an account balance in his employer's money-purchase pension plan of $100,000. The plan has a 2-to-6-year graded vesting policy. He has been a participant for 3½ years and has worked for the company for 5 years. Assuming the plan permits loans, what is the maximum loan that Jamal could take from the plan? A) $20,000 B) $30,000 C) $40,000 D) $50,000

The correct answer is (C). The maximum loan an individual can take is the lesser of $50,000 or 50 percent of his or her vested account balance. In this case, Jamal is 80 percent vested because he has worked for 5 years, which equates to 80 percent on the 2-to-6 -year graded schedule. Half of 80 percent of $100,000 equals $40,000.

Joe's employer offers a defined-benefit pension whose payout is a function of the 3 highest years of earnings. Jim currently has a 3-year average earnings of $80,000, and his accrued monthly pension is $2,500. If Joe's employer wishes to add an incidental whole life insurance policy to the pension plan, what is the maximum death benefit that can be offered to Joe? A) $80,000 B) $240,000 C) $250,000 D) $2,500,000

The correct answer is (C).A qualified pension plan is limited in the amount of life insurance it is able to purchase with plan assets. Either the 25% / 50% test or the 100-to-1 ratio test must be passed for the insurance to be considered incidental. Since we have no information about aggregate employer contributions, it isn't possible to apply the 25% / 50% test. In using the 100-to-1 ratio test, the maximum death benefit is 100 x $2,500 = $250,000.

Colin owns and operates Colin's Creative Coaching (3C), a sole proprietorship. 3C sponsors a 401(k)/profit-sharing plan. During the year, Colin had net income of $200,000 and half his self-employment taxes were $10,000. Assuming Colin is over the age of 50, what is the maximum amount that Colin and his company can contribute on his behalf to the plan for 2022? A) $38,000 B) $61,000 C) $65,000 D) $67,500

The correct answer is (C).Colin can contribute a profit-sharing, 401(k), and catch-up contribution. $200,000 Net Income ($10,000) Less ½ SE Tax $190,000 Net SE Income × 0.20 0.25 ÷ 1.25 $38,000 $20,500 401(k) Contribution + $6,500 Catch-up $65,000 Total

On January 15, Year 1, Blake, a senior vice president for Acme Corporation, is granted 20,000 ISOs at an exercise price of $10. On February 6, Year 2, he exercises all his options when the price of Acme stock is $27. When is the earliest Blake can sell the ISO shares and avoid a disqualifying disposition? A) January 16, Year 3 B) January 16, Year 4 C) February 7, Year 3 D) February 7, Year 4

The correct answer is (C).For a qualifying disposition, he must wait two years from the date of grant and one year from the date of exercise.

In which of the following scenarios would a qualified distribution from a Roth IRA have been made? A) A 60-year-old opens her first Roth IRA and 3 years later takes a distribution. B) A 40-year-old takes $10,000 from a 10-year-old Roth IRA to help pay for a second home. C) A 35-year-old becomes totally disabled and distributes $100,000 from a 6-year-old Roth IRA. D) A 55-year-old with a 30-year-old Roth distributes $5,000 to pay his daughter's preschool tuition.

The correct answer is (C).In order to be considered a qualified distribution, a Roth IRA must be open for at least 5 years and one of the following must be true: the account holder is at least 59 ½ years old, the account holder is dead and the beneficiary receives the distribution, the account holder is disabled, or the account holder is taking out up to $10,000 to help pay for a first-time home purchase. Answer (A) is incorrect because the Roth has not been open for 5 years. Answer (B) is incorrect because the distribution is for a second home. Answer (D) is incorrect because paying for preschool education is not considered a qualified distribution.

Seth and Morgan were just married, but Seth does not work. They are both 25 years old. Morgan supports Seth with funds from her trust fund, investment income of $12,000, and her part-time earnings of $7,000 this year. If Morgan contributes $6,000 to her Roth IRA this year, how much can Seth contribute to his traditional IRA? A) $0 B) $250 C) $1,000 D) $6,000

The correct answer is (C).Seth and Morgan have combined earned income of $7,000 for the current year, which limits his ability to contribute to his IRA. $7,000 − $6,000 = $1,000.

Bob, who is 58 years old, makes his first contribution to a Roth IRA in December of Year 1. What is the first year in which he could receive a qualified distribution from a Roth IRA? A) Year 2 B) Year 3 C) Year 5 D) Year 6

The correct answer is (D). A qualified distribution can only occur after a 5-year period and is made on or after the date on which the owner attains age 59½, made to a beneficiary or the estate of the owner on or after the date of the owner's death, attributable to the owner's being disabled, or made for a first-time home purchase. The 5-year period begins at the beginning of the taxable year of the initial contribution to a Roth IRA. The 5-year period ends on the last day of the individual's fifth consecutive taxable year beginning with the taxable year described in the preceding sentence. Therefore the first year in which a qualified distribution could occur is Year 6.

At the beginning of this year, Sophia contributed $3,000 to her HSA. She invested that contribution into stocks and, by the end of the year, her account balance grew to $4,000. Which of the following statements correctly describes the tax treatment of this contribution and subsequent gain? A) The full $4,000 will be included in her gross income but she will be able to make a $3,600 above-the-line deduction for the current tax year. B) $3,600 will be included in her gross income but will be negated by a $3,600 above-the-line deduction; the remaining $500 will be subject to capital gains tax treatment for the current tax year. C) She will be able to make a $3,000 above-the-line deduction and the $1,000 gain will not be included in her gross income or subject to capital gains tax treatment for the current tax year. D) She will be able to make a $3,000 above-the-line deduction and the $1,000 gain will be subject to capital gains tax treatment for the current tax year.

The correct answer is (C).Sophia's $3,000 contribution will be considered an above-the-line deduction from her gross income. The subsequent gain will not be included in her gross income or subject to capital gains taxes.

Colin is 35 years old and inherits an IRA from his mother, who died at age 60. Which of the following statements regarding his options for the inherited IRA is correct? A) Colin does not have to take distributions until his mother would have been 72 years old. B) Colin can rollover the IRA into his own IRA. C) Colin can take out the entire distribution within 10 years and avoid all penalties. D) Colin must take distributions over his single life expectancy.

The correct answer is (C).Statements (A) and (B) are options available to surviving spouses, but not to a child. Option (D) is no longer available under the SECURE Act.

All of the following statements regarding stock bonus plans are correct EXCEPT: A) Stock bonus plans allow for the current deductibility of non-cash contributions. B) The required repurchase option for a stock bonus plan can create potential cash-flow issues in the future. C) Stock bonus plans are generally as cost efficient to operate as profit-sharing plans or money-purchase pension plans. D) The eligibility for a stock bonus plan could be that the person must be aged 20 and have given 6 months of service.

The correct answer is (C).Statements (A), (B), and (D) are true. Statement (C) is incorrect as the cost to value stock for a private stock bonus plan increases the administrative costs in comparison to profit-sharing or money-purchase plans.

There are additional tax advantages, beyond the mismatch of income and deduction, for the establishment of an employee stock ownership plan (ESOP). One of these is nonrecognition of gain treatment. To obtain nonrecognition of gain treatment, what percent of company stock must the ESOP own after the transaction? A) 85% B) Over 50% C) 30% D) 25%

The correct answer is (C).The ESOP must own at least 30 percent of the stock immediately after the sale.

Which of the following is the most common defined-benefit plan funding formula for large companies? A) The flat amount formula B) The flat percentage formula C) The unit credit formula D) The excludable amount formula

The correct answer is (C).The most common funding formula for a defined-benefit plan is the unit credit formula, which provides higher benefits for longer-term employees. The flat amount and flat percentage formulas are less common, and the formula depicted in statement (D) does not exist.

Parker, aged 51, earns $300,000 annually from Infinity, which sponsors a SIMPLE and matches all employee deferrals at 100 percent for up to a 3 percent contribution. What is the maximum total contribution to Parker's account in 2021, including both employee and employer contributions? A) $21,400 B) $22,500 C) $24,900 D) $25,500

The correct answer is (D Parker can defer up to $16,500 (or $13,500 + $3,000) for 2021 because he is over 50. The match for Parker is 3 percent of his compensation, or $9,000 (that is 3% × $300,000). The maximum contribution to Parker's SIMPLE is $25,500 (or $13,500 + $3,000 + $9,000).

Alfred has worked for CJD, a large manufacturer, for the last 20 years and is a participant in CJD's defined-benefit plan. Alfred is concerned about the company's financial difficulties and is worried that management at CJD might modify his future benefits and cause his expected benefits at retirement to be reduced. Which of the following laws is designed to prevent that from occurring? A) Coverage B) Non-discrimination C) Anti-cutback D) There are no laws that prevent an employer from modifying future benefits, even benefits in a defined-benefit plan.

The correct answer is (D). • Future benefits can be modified by employers.• Coverage is designed to ensure that a minimum number of non-highly compensated employees are benefiting under the plan.• Non-discrimination rules do not prevent loss of benefits.• Anti-cutback is a rule prohibiting a plan sponsor from amending a plan such that the accrued benefit of an employee is decreased or reduced by such an amendment or change.

All of the following are requirements of employers wishing to establish a SEP IRA EXCEPT: A) Contributions can be made for all qualifying employees who have reached the age of 21. B) Contributions can be made for all qualifying employees who have performed services in 3 of the past 5 years. C) Contributions can be made for all qualifying employees who have earned a minimum compensation of $650 in the past year. D) Contributions to the plan must be made no later that 12/31 of the current plan year.

The correct answer is (D). (A), (B) and (C) are correct. (D) is false because SEPs can be funded by the tax filing deadline of the employer, and contributions are not required to be made by year-end.

All of the following can be invested in within an IRA EXCEPT: A) Cash B) Stocks and bonds C) US gold, silver, and platinum coins D) Other collectibles, including Krugerands and maple leaf coins

The correct answer is (D). (A), (B), and (C) are all correct. (D) is false, because foreign coins and collectibles, in general, are not permitted investments in IRAs.

All of the following are exceptions to the 10% early withdrawal penalty EXCEPT: A) Death B) Attainment of age 59½ C) Substantially Equal Periodic Payment Plans D) Payment of health insurance premiums by employed individuals

The correct answer is (D). (A), (B), and (C) are correct. (D) is false because payment of health insurance premiums is only permitted for unemployed individuals, not employed individuals.

All the following types of income are considered earned income for the purposes of making a traditional IRA contribution EXCEPT A) income from a general partnership. B) W-2 income. C) self-employment income. D) alimony resulting from a divorce agreement signed in 2021.

The correct answer is (D). Alimony resulting from a divorce agreement signed this year will count as earned income, but divorce agreements signed after 2018 (2017 TCJA) will not count as earned income. The other options qualify as earned income for purposes of IRAs.

ISOs are a common benefit for executives. All of the following statements are correct regarding ISOs EXCEPT: A) ISOs can only be granted to an employee of the corporation issuing the ISOs. B) The exercise of the ISO is limited to a 10-year period. C) To the extent that the aggregate fair market value of stock with respect to which ISOs are exercisable for the first time by any individual during any calendar year exceeds $100,000, such options shall be treated as NQSOs. D) To qualify as an ISO, the executive must hold the stock for either two years from the grant of the ISO or one year from the date of exercise of the ISO.

The correct answer is (D). All statements are correct except (D). The statement in answer (D) was changed from "the greater of" to "either."

Under the prohibited transaction statutes, a disqualified person can refer to any of the following people EXCEPT A) an officer or director of the sponsoring company. B) a 10 percent shareholder of the sponsoring company. C) the spouse of a 51 percent owner of the sponsoring company. D) an employee of the sponsoring company with less than 1 year of service.

The correct answer is (D). As long as an employee of the sponsoring company is not an officer, director, 10 percent shareholder, or highly compensated employee, then the employee will not fit the requirements to be a disqualified person. Spouses of shareholders with more than 50 percent ownership may also be disqualified people.

David has worked as a professor at Top-Notch University for the last 30 years. He has never deferred any money into his 457(b) plan. He has maxed out his 403(b) plan. He will attain his normal retirement age under the 457(b) plan in 2022. How much can David contribute to his 457(b) plan for 2021? A) $0 B) 19,500 C) $26,000 D) $39,000

The correct answer is (D). Because David has never contributed to his 457(b) plan, he has unused prior deferrals. He may therefore defer the maximum for 2021 ($19,500) plus his maximum unused prior deferrals ($19,500) for a total maximum contribution of $39,000.

Charles, a single 29-year-old CEO of a technology start-up company, earns a $2 million base salary with a $400,000 bonus. He is not a participant in any retirement plans at work. What is the maximum deductible IRA contribution Charles can make for the 2022 tax year? A) $0 B) $1,000 C) $5,500 D) $6,000

The correct answer is (D). Because he is not an active participant in a retirement plan, he can contribute and deduct the maximum amount of $6,000 for the 2022 tax year.

A company has just made a contribution to a non-qualified deferred compensation plan on behalf of an employee. That contribution will have a substantial risk of forfeiture for the next 15 years. Which of the following statements with regard to the tax treatment of the contribution is correct? A) It is a deductible expense to the company. B) The employee must pay ordinary income taxes on the contribution. C) The employee must pay capital gains taxes on the contribution. D) The employee must pay payroll taxes on the contribution.

The correct answer is (D). Contributions to a deferred compensation plan are subject to payroll taxes but not ordinary income taxes or capital gains taxes. They are also not deductible by the employer.

Urban, LLC, sponsors a profit-sharing plan that requires employees to complete one year of service and be 21 years of age before entering the plan. The plan also excludes all commissioned salespeople and all other allowable exclusions under the IRC. Which of the following employees could be excluded? I. Jack, aged 20, who works in administration. He has been with the company for 32 months.II. Jen, a commissioned salesperson working in the Houston office. She is 37 years old and has been with the company for 3 years.III. John, a 39-year-old who works as the lead foreman in the company factory. John has been with the company for 12 years and is covered under a collective bargaining agreement. A) I only B) III only C) I and II D) I, II, and III

The correct answer is (D). Each of these employees can be excluded from the plan. Jack does not meet the age requirement. Jen can be excluded because she is a commissioned salesperson. John is excluded because he is covered under a collective bargaining agreement.

Which of the following is an advantage of a qualified plan over a non-qualified plan? A) Lower administrative costs B) More useful for attracting executives and other highly compensated employees C) Higher contribution limits D) More favorable tax treatment for employees

The correct answer is (D). Qualified plans provide the most favorable tax treatment for employees. However, their lower contribution limits make them less appropriate or useful for attracting highly compensated executives. Moreover, they generally have higher administrative costs.

Participation in which of the following plans will not be considered active participation for determining the deductibility of traditional IRA contributions? A) A tax-sheltered annuity B) A simplified employee pension C) A SIMPLE D) A 457 plan

The correct answer is (D). The 457 plan is not considered a retirement plan under the IRA deductibility rules.

The Actual Deferral Percentage (ADP) test is one of the tests with which some qualified plans must comply. Which of the following plans must generally comply with the ADP test? I. Profit-sharing plans without a CODAII. Safe harbor 401(k) plans A) I only B) II only C) Both I and II D) Neither I nor II

The correct answer is (D). The ADP test generally only applies to non-safe harbor 401(k) plans. It does not apply to profit-sharing plans without a CODA.

All of the following statements concerning the income limits imposed for determining whether one is eligible to receive a deduction for contributions made to a traditional IRA are correct EXCEPT: A) The income limits only apply to taxpayers who are covered by a qualified retirement plan. B) The income limits are higher for Roth IRAs than for traditional IRAs. C) Taxpayers whose income is over the limit may still make non-deductible contributions to traditional IRAs. D) Marital filing status does not affect IRA income limits.

The correct answer is (D). The IRA income limits for married-filing-jointly taxpayers are about twice as high as the limits for single filers. All of the other statements are correct.

Parker is a highly skilled salesperson at Byberry, which is a 30-year-old company that has grown significantly in terms of revenue and product offerings. It sponsors a pension plan that provides a benefit of 2 percent times the years of participation times the average of the final 3 years of salary. Parker has worked for Byberry for the last 30 years and earned $200,000 2 years ago, $150,000 last year, and $250,000 this year. If he is retiring this year, how much should he expect to receive as a pension benefit? A) $200,000 B) $185,000 C) $160,000 D) $120,000

The correct answer is (D). The average salary over the last 3 years is $200,000. The benefit equals 60 percent of $200,000, or $120,000.

Robin, who is 55 years old, runs a local restaurant in New Orleans. Several high school kids work for her part-time. Robin's mom, Deirdre, works there full-time. Which of the following plans makes the most sense for Robin if she earns about $120,000 and does not want to spend too much on a retirement plan? A) A money-purchase plan B) A tandem plan C) A cash-balance plan D) A 401(k) profit-sharing plan

The correct answer is (D). The first three options require annual funding, which generally is not the best choice for a small business. Option (D) permits Robin to save over $50,000 per year, or almost half of her income, between the salary deferral, the catch-up, and the profit-sharing contribution. In addition, she has no requirement to fund it all, as the plan is discretionary.

Dr. Durr contributes $85,000 to one of his university's retirement-type plans. The HR employee who helped him set up this plan told him it was a non-qualified plan only available to highly compensated and top management employees. To what type of plan did he contribute the funds? A) A tax-sheltered annuity B) A public 457 plan C) A private 457 plan D) An ineligible 457 plan

The correct answer is (D). The ineligible 457(f) plan is a non-qualified deferred-compensation plan only offered to highly compensated or top management employees.

Drake has worked for GT for the last 20 years and has been a participant in its defined-benefit plan. In the last 10 years, his salary has changed significantly. Over that time, his compensation was as follows: Year 1: $300,000 Year 2: $245,000 Year 3: $200,000 Year 4: $400,000 Year 5: $225,000 Year 6: $240,000 Year 7: $233,000 Year 8: $210,000 Year 9: $250,000 Year 10: $290,000 In 2022, what is the most that he could receive as a pension payment? A) $330,000 B) $288,333 C) $280,000 D) $245,000

The correct answer is (D). The maximum distribution from a defined-benefit plan in 2022 is $245,000 (or the average of the 3 highest consecutive years of compensation, whichever is lower).

All of the following statements regarding age-based profit-sharing plans are correct EXCEPT: A) An age-based profit-sharing plan provides greater benefits to the older plan participants. B) Younger plan participants in an age-based profit-sharing plan usually receive the minority of the profit-sharing plan allocation. C) Non-discrimination testing in age-based profit-sharing plans is based on benefits and not contributions. D) An age-based profit-sharing plan provides a greater benefit to those participants whose earnings exceed the Social Security wage base and who are older than 50 years old.

The correct answer is (D).Age-based profit-sharing plans provide increased benefits to older participants regardless of whether their wages are above the Social Security wage base. All the other statements are true.

Which of the following types of income is considered earned income for the purposes of making a Roth IRA contribution? A) Dividends B) Rental income C) Capital gains D) Partnership income from a law firm

The correct answer is (D).Dividends and capital gains are investment income. Rental income is passive. Partnership income is considered earned income if it is from active engagement of the business.

Axe Company sponsors a 401(k) profit-sharing plan with no employer match, but the company did make noncontributory employer contributions because the plan was top-heavy. Jack quit today after 6 years of working for Axe and has come to you to determine how much of his retirement balance he can take with him. The plan uses the least generous graduated vesting schedule available. What is Jack's vested account balance if he has been a participant for 60 months? EMPLOYER EMPLOYEE CONTRIBUTIONS $9,000 $12,000 EARNINGS $4,000 $5,000 A) $17,000 B) $24,800 C) $27,400 D) $30,000

The correct answer is (D).Jack is entitled to 100 percent of his contributions and the earnings on those contributions. The employer contributions, which were not matching contributions, will follow the least generous graduated vesting schedule for a top-heavy plan. The least generous graduated vesting schedule is a 2- to-6 year graduated vesting schedule for a 401(k) plan. At 6 years of service, Jack would be 100 percent vested in the employer contributions and the earnings on the employer contributions. Thus, Jack's vested account balance is $30,000.

Marleen is a 52-year-old participant in the XYZ cash-balance plan. She has been a participant in the plan for the last 20 years. XYZ, which has over 10,000 employees, is having financial difficulty and Marleen is concerned about the security of her pension. Which of the following is correct? A) The cash-balance plan assets may include up to 25 percent of XYZ stock. B) The cash-balance plan formula cannot be lowered in the future for current participants, but it could be changed for new participants. C) The cash-balance plan and its benefits are fully protected by the Pension Benefit Guaranty Corporation (PBGC). D) Termination of the plan may affect vesting for some employees, but Marleen's vesting will not be affected.

The correct answer is (D).Option (A) is incorrect because up to 10 percent of plan assets can be invested in the plan sponsor's stock. Option (B) is incorrect as the plan sponsor can modify future benefits, but cannot modify benefits that are already accrued. Option (C) is incorrect because the PBGC does not fully protect all benefits; it only protects benefits up to an annual limit. Option (D) is correct; termination accelerates vesting, but based on Marleen's tenure, she is fully vested already.

Which of the following is a characteristic of pension plans that does not apply to profit-sharing plans? A) Separate accounts B) In-service withdrawals for select employees, plan permitting C) Limited investment in life insurance D) Mandatory funding

The correct answer is (D).Pension plans require mandatory funding, but profit-sharing plans do not. In-service withdrawals may be offered in a profit-sharing plan. A pension plan may allow in-service withdrawals for employees over the age of 62. Both pension and profit-sharing plans have restrictions on the amount of life insurance that can be purchased within the plan.

Jack and Jill own a successful engineering company that sponsors a 401(k) plan that requires standard eligibility. Sam, Tom, and Pat are the only other employees. These employees are between the ages of 25 and 29 and have been with the company for a couple of years. Jack and Jill each have salaries of $200,000, while their employees have salaries ranging between $28,000 and $30,000. Jack and Jill both defer $10,000 each. Sam, who is Jack and Jill's son, earns $30,000 and defers $6,000 into the 401(k) plan. Tom, who makes $28,000, defers $2,800, while Pat does not defer anything into the 401(k) plan. Which of the following statements is correct? A) The ADP for the non-highly compensated employees (NHCEs) is 6.67 percent. B) Jack and Jill are the only highly compensated employees (HCEs) C) If the plan failed the ADP test, the issue could be solved by providing a qualified matching contribution (QMC) to all five employees. D) If the company hired a new employee, it would not increase the amount that Jack and Jill can defer during the first year of the employee's employment.

The correct answer is (D).Sam is highly compensated through family attribution. The ADP of the NHCEs is 5 percent, not 6.67 percent. Statement (C) is not correct as a QMC would only go to NHCEs.

Defined-benefit plans and cash-balance plans are both pension plans. However, they are significantly different types of plans. Which of the following statements is true? A) A cash-balance pension plan benefits younger and older employees equally. B) Both plans have hypothetical accounts for each plan participant. C) Both plans use the same vesting schedules regardless of whether the plan is top heavy. D) Both plans can provide for lump-sum benefits upon termination and/or full retirement age.

The correct answer is (D).Statement (D) is true. Statement (A) is false because cash-balance pension plans usually benefit younger participants because of the guaranteed contribution rate and the guaranteed earnings rate. Statement (B) is false because a defined-benefit plan promises a defined benefit at the participant's retirement. It does not have a hypothetical account like a cash-balance plan. Statement (C) is false because cash-balance plans must use the 3-year cliff vesting.

All of the following steps are included in the Taxonomy of Qualified Plan Selection EXCEPT: A) Confirm that the plan sponsor willing to meet Qualified Plan requirements(B) Prepare an Employee Census B) Prepare an Employee Census C) Choose between mandatory funded pension plan and discretionary funded profit sharing plans D) Choose the plan that bests meets the objectives of the employees of the organization

The correct answer is (D).The 4th step in the Taxonomy of Plan Selection is to choose the plan that best meets the objectives of the OWNER of the organization.

The High-End Preparatory Boarding School has a residence on its property for the principal. Jacques, who is the principal, and his family live in the residence, which is provided as part of his job. The school's board of directors consider it important that the principal be close to the school in case any issues arise. Jacques also eats at the school cafeteria, typically two meals each school day. Which of the two benefits must be included in Jacques' gross income for tax purposes? I. LodgingII. Meals A) I only B) II only C) Both I and II D) Neither I nor II

The correct answer is (D).The meals and lodging are provided for the convenience of the employer. After 2017, the TCJA 2017 subjects the meals for the convenience of the employer to the 50-percent deduction limit.

Colleen receives a lump-sum distribution of employer securities (1,000 shares) from her stock bonus plan in Year 1 worth $140,000. The net unrealized appreciation (NUA) for the stock equals $110,000 at the time of the distribution. Colleen sells half of the shares 4 years after she received them as a distribution from the qualified plan. She receives proceeds of $100,000 from the sale. How much is Colleen's capital gain? A) $55,000 B) $65,000 C) $75,000 D) $85,000

The correct answer is (D).The tax basis for the 1,000 shares is $30,000, the difference between the NUA and the proceeds. If she sells half the shares for $100,000, then the basis for that half equals $15,000. The long-term capital gain equals $85,000.

Marleen is 55 years old and makes $100,000 per year. She saves $12,350 per year and pays FICA expense of $7,650. She wants to assume that any other expense reductions will be offset by increased retirement expenditures, such as health care and vacations. Based on these assumptions, her WRR equals 80%. $100,000 = 100.00% of salary in % terms (12,350) = (12.35%) less: current savings in % terms (7,650) = (7.65%) less: payroll taxes in % terms (not paid in retirement) $80,000 = 80.00% wage replacement ratio in % terms Marleen lives off about 80% of her pre-retirement income. Therefore, she will need about $80,000 in income during retirement. The remaining 20% or $20,000 is currently spent on retirement savings and FICA.

Top Down Approach example

Almost half of American have less than $25,000 in savings and one in four have less than $1,000. True or False?

True

Expenses in Retirement/Pre-Retirement Income

Wage Replacement Ratio (WRR)

The period of time a person is expected to be in the work force. Period during which one saves and accumulates for retirement. Generally 30-40 years, but has been decreasing due to advanced education and early retirement.

Work Life Expectancy (WLE)

savings rate

a percentage of income that a person is saving.


Related study sets

Chapter 09 Examination and treatment Areas

View Set

NUR3010: CHAPTER 8- PREPU QUIZZES

View Set

Evolve: Cardiovascular, Perfusion EAQ, Cardiovascular EAQ

View Set

Chapter 18: Peri-Op Nursing Care

View Set